plz help me with the attachment below

Plz Help Me With The Attachment Below

Answers

Answer 1

Answer:

57

Step-by-step explanation:

( 27 + 19 + 41 + 51 + 29 + 35 + k ) / 7 = 37

202 + k = 37 x 7

202 + k = 259

k = 259 - 202

k = 57

Answer 2

Answer:

K. = 57

Step-by-step explanation:

Plz Help Me With The Attachment Below

Related Questions

Eight hundred tickets are sold for a play. Thirty-five percent of those tickets were sold in advance. Which equation can be used to find the number of tickets sold in advance? StartFraction 800 divided by 8 Over 35 divided by 8 EndFraction = StartFraction 100 Over 4.375 EndFraction StartFraction 100 divided by 8 Over 800 divided by 8 EndFraction = StartFraction 12.5 Over 100 EndFraction StartFraction 35 times 8 Over 800 times 8 EndFraction = StartFraction 280 Over 6400 EndFraction StartFraction 35 times 8 Over 100 times 8 EndFraction = StartFraction 280 Over 800 EndFraction

Answers

Answer:

0.35(800)=x

x=number of tickets sold in advance

Step-by-step explanation:

Answer:

Divide 35÷100 which equals 0.35 then multiply 0.35×800 which equals 280 so the answer is 280 so the equation is 0.35×800=280.

Step-by-step explanation:

Louise owns a hot dog stand. The amount of money she collects from selling hot dogs each day can be found by using the equation y=3.5x What does 3.5 from the equation represent in the situation?

Answers

Answer:

3.5 represents the cost of each hotdog ($3.50).

Step-by-step explanation:

In y = 3.5x, the x represents the number of hotdogs and the y represents the total amount you have to pay for those hotdogs. The 3.5 represents the cost of each hotdog.

3.5 (3.50$)is how much each hotdog cost

Evaluate the expression shown below and write your answer as a fraction in simplest form.

Answers

5/40 + 9/40 = 14/40
7/20

Answer:

7/20 if you want you answer in decimal form that would be 0.35

How do you call a function that have a power higher than 1?

Answers

the power in the function
The power is in the function

Leon used the number line below to show -5 + 4. explain what is wrong with his model.

Answers

Well since he used -5, then it should be +5.
Step by step:
Hope that help a little I guess.

When you add 6 to the product of a number and 10, the result is 36. What is the number?
A) 3
B) 6
C) 2
D) 4

Answers

Answer: the number is 3.

Step-by-step explanation:

(x × 10) + 6 = 36

[(3) × 10] + 6 = 36

Answer:

3

Step-by-step explanation

First let's create an equation

Let x = number

Product is outcome of multiplication

The product of 10 and a number can be written as 10x

Were supposed to add 6 to the product and that should equal 36

So the equation is 10x + 6 = 36

We now solve for x using basic algebra

10x + 6 = 36

Subtract 6 from both sides

10x = 30

Divide both sides by 10

X = 3

Which of the equations is graphed below?

Answers

Answer:

D

Step-by-step explanation:

Slope of -2 that's going through y=1

so y=-2x+1

d is the correct answer bc if you plug in 0 you get 1 which is a point on the graph

How would you do this?

Answers

it would be 6.0x4 because the square is 6.0 on each side
The answer is 6.0 x 4

Help me please! It’s due in like 10 minutes and I’m barely on the 8th question !!!

Answers

Answer:A

Step-by-step explanation:campbella helped

Answer:

A

Step-by-step explanation:

6(5-3x) -5(3x+2)

multiply each expression by the number outside the bracket

6×5=30,6×-3x=-18x -5×3x=-15x,-5×+2=-10

30-18x-15x-10

group the like terms

30-10-18x-15x

20-33x

y varies inversely as the square of x
y=1.5 when x=8
Find y when x=5

Answers

Answer:

3.84

Step-by-step explanation:

Given that , y varies inversely as the square of x . Mathematically we can write this statement as ,

[tex]\implies\rm y \propto \dfrac{1}{x^2}[/tex]

Let k be the constant . Therefore ,

[tex]\implies\rm y = k \dfrac{1}{x^2}[/tex]

When y = 1.5 and x = 8 :-

Plug in the respective values ,

[tex]\implies\rm y = k \dfrac{1}{x^2} \\\\\implies\rm 1.5 = k \times \dfrac{1}{8^2} \\\\\implies\rm k = 1.5 \times 64 \\\\\implies\rm k = 96[/tex]

When x = 5 :-

[tex]\implies\rm y = k \dfrac{1}{x^2} \\\\\implies\rm y = 96 \times \dfrac{1}{5^2}=\dfrac{ 96}{25} \\\\\implies\rm\boxed{ y = 3.84 }[/tex]

Answer: 3.84

hope it helped!

Give correct answer and I will give you branlist

Answers

Hello!

Space = 5 cm + 10cm + 10cm + 20cm + 22cm + 30cm => space = 15cm + 10cm + 20cm + 22cm + 30cm => space = 25cm + 20cm + 22cm + 30cm => space = 45cm + 22cm + 30cm => space = 67cm + 30cm => space = 97cm²

Good luck! :)

Answer:

97cm square

Step-by-step explanation:

Grace adopted a kitten, Fluff, from the animal shelter. Fluff weighed 3.5 lbs. when she was adopted and has gained weight at a rate of 1.5 lbs. per week. Which equation could be used to determine after how many weeks, x, Fluff would weigh 12.5 lbs.?



3.5x+12.5=1.5

3.5x+1=12.5

3.5+1.5x=12.5

3.5−1.5x=12.5

Answers

Answer:

C: 3.5+1.5x=12.5

Step-by-step explanation:

i think this is correct because in the question they use the word Gain which means you would add. But also this answer is correct because the kitten had gain 1.5 pounds PER WEEK which make 1.5x

so your answer is 3.5+1.5x=12.5

Hope this helps have a good day

and if correct pls mark me brainliest

thank you bye

Answer:

3.5 +1.5x = 12.5 (third option top- down)

Step-by-step explanation:

given : weighted 3.5 lbs. then gained 1.5 lbs. per week ended up 12.5 lbs.

translate:             3.5                   +      1.5         *        x         =           12.5

Evaluate the expression shown below and write your answer as a fraction in simplest form.

Answers

Answer:

[tex]43/100[/tex]

Step-by-step explanation:

First, make both of the denominators into 100.

Then, 3/10 would be 30/100.

Finally, you can add them to get 43/100.

43/100
Make both denominators into 100 so that means 3/10 would be 30/100 then you add both so 30/100+ 13/100 only adding the numerators so it will equal 43/100

In which line is an error first made when solving the equation below? -18 = 19z -9
Line One = z -9
Line Two = 11 = z
Line Three = z = 11
* Choose one of the following, A, B, C, D.*
A) no mistake was made
B) line 3
C) line 2
D) line 1

Answers

It’s line 3. Hope that helps :)

does this make sence to anyone

Answers

What the dude above you said
The answer would be 490 one because the surface area is just the area of the whole figure so you’d find the area of each figure and add them all up

In the figure shown, line AB is parallel to line CD.

Part A: What is the measure of angle x? Show your work. (5 points)

Part B: Explain how you found the measure of angle x by identifying the angle relationships that you used along the transversal. (5 points)

Answers

Answer:

Part A

x is 46°

Part B

Alternate angles are angles that are in relatively opposite locations relative to a transversal

Step-by-step explanation:

Part A

∠DRP  = 110° (Given)

∠QPA = 64° (Given)

∠QPR =

Given that AB is parallel to CD, we have;

∠DRP is congruent to ∠APR (Alternate angles to a transversal RP of parallel lines AB and CD)

Therefore, ∠APR = 115°

∠APR = ∠QPA + ∠QPR (Angle addition postulate)

∴ 115° = 64° +  ∠QPR

∠QPR = 110° - 64° = 46°

x = 46°

Part B.

Given that AB is parallel to CD, the lines common (that intersects) both lines are the transversal lines

The angles formed between the parallel lines and the transversal lines have special relationships based on their position with respect to each other

In the question, the angle 110° given between CD and the transversal RP, is found to at an alternate position to the angle ∠APR between the same transversal RP and AB and given that alternate angles are always equal, angle ∠APR is therefore also equal to 110°.

X would equal 46 degrees

Juan runs in place for 2 5/6 minutes on Monday. He runs in place for 1 1/2 times as long on Tuesday.

Juan draws this model to represent the number of minutes he runs in place on Tuesday.

How many minutes does Juan run in place on Tuesday?

Enter your answer as a mixed number in simplest form by filling in the boxes.

Answers

Answer:

12miles

Step-by-step explanation:

wink :)

simplify

(x^a.x^b)^a-b*(x^b.x^c)^b-c*(x^c.x^a)^c-a

step by step..

Answers

Answer:

= x(a+b)(a-b) . x(b+c)(b-c) .x(c+a)(c-a)

Answer:

x(a+b)(a-b) . x(b+c)(b-c) .x(c+a)(c-a)

Hope it helped!

Please help! I’m confused.

Answers

Step-by-step explanation:

We need to use the formula :-

2 (l + b) × h ---> For Lateral surface area

2(30+30) h = 7200

2×60×h = 7200

120 × h = 7200

h = 7200/120

h = 60 cm

Now, volume = l×b×h

= 30×30×60

= 54000 cm³ is the required answer.

Hope it helps! :D

Answer:

54000cm³

Step-by-step explanation:

7200=4*30x,so x=60

In other words, the length of the vertical side is equal to 60cm

The volume inside the kite=60*30*30=54000cm³

From an 18-pound cut of meat, Mr. Rand cuts off steaks weighting 1 pound 15 ounces, 3 1/8 pounds, and 4 1/2 pounds. He divides up the rest into 9-ounces patties and sells them at 3 for $3.12. How much money does he receive for the patties?

Answers

Answer is in the picture

The amount of money received by him will be $15.60.

What is an expression?

Expression in maths is defined as the collection of the numbers variables and functions by using signs like addition, subtraction, multiplication, and division

The amount will be calculated as follows:-

The cut off amount = 18 - ( 1(15/16) -3(1/8)-4(1/2) = 8(7/16)

Now 16 ounces make a pound so divide by 16 we get:-

(135/16) ÷ ( 9 / 16 ) = ( 135/16 ) x ( 16 / 9 ) = 15

Dividing 15 by 3 we get 5 packs of 3 patties for $3.12

So amount = 3.12 x 5 = $15.60

Therefore the amount of money received by him will be $15.60.

To know more about Expression follow

https://brainly.com/question/723406

#SPJ2

HELP PLEASE!!! i will give brainly! +10 points

Suppose each cube in this right rectangular prism is a -inch unit cube.



What are the dimensions of the prism?
What is the volume of the prism?

Answers

Answer:

1 number 1

Step-by-step explanation:

1 because of 1 of 1

Answer:

1000  hope its right

Step-by-step explanation:

8 · Find the volume of the rectangular prism with the dimensions 11 by 9 by 3. 23 in 297 in 318in 159in I think the answer is c 318 in. geometry A rectangular prism has a length of 4.2 cm, a width of 5.8 cm, and a height of 9.6 cm.

Is the triangle obtuse, acute, equilateral or right?

Answers

9514 1404 393

Answer:

  obtuse

Step-by-step explanation:

The law of cosines tells you ...

  b² = a² +c² -2ac·cos(B)

Substituting for a²+c² using the given equation, we have ...

  b² = b²·cos(B)² -2ac·cos(B)

We can subtract b² to get a quadratic in standard form for cos(B).

  b²·cos(B)² -2ac·cos(B) -b² = 0

Solving this using the quadratic formula gives ...

  [tex]\cos(B)=\dfrac{-(-2ac)\pm\sqrt{(-2ac)^2-4(b^2)(-b^2)}}{2b^2}\\\\\cos(B)=\dfrac{ac}{b^2}\pm\sqrt{\left(\dfrac{ac}{b^2}\right)^2+1}[/tex]

The fraction ac/b² is always positive, so the term on the right (the square root) is always greater than 1. The value of cos(B) cannot be greater than 1, so the only viable value for cos(B) is ...

  [tex]\cos(B)=\dfrac{ac}{b^2}-\sqrt{\left(\dfrac{ac}{b^2}\right)^2+1}[/tex]

The value of the radical is necessarily greater than ac/b², so cos(B) is necessarily negative. When cos(B) < 0, B > 90°. The triangle is obtuse.

Obtuse Bc Ik it is so ye lol

Pls help
Spam answers will be reported
Give a good explanation

Answers

Answer:

X = 20

The picture explains my answer

Answer:

The answer is X = 20

A container is in the shape of a triangular pyramid and has a height of 10 inches. The area of the base of the container is 12 square inches.

What is the volume of the figure in cubic inches?

40 cubic inches
120 cubic inches
60 cubic inches
360 cubic inches

Answers

40 cubic inches is the
60- (10x12)/3 this should be right

Identify the initial value and rate of change for the graph shown

A coordinate plane graph is shown. A line passes through the y-intercept at 1 and through the point 5 comma 4.

Initial value: 1, rate of change 3 over 5.
Initial value: 3 over 5., rate of change: 0
Initial value: 1, rate of change 5 over 3.
Initial value: 5 over 3., rate of change: 0

Answers

initial value: 1. rate of change: 3 over 5.
Initial value:1,rate of change 3 over 5

Rewrite 24.5 as a mixed number in lowest terms.



Rewrite 0.85 as a fraction in lowest terms.

Answers

Answer:

49/2, 17/20

Step-by-step explanation:

24.5=245/10=245÷5/10÷5=49/2

0.85=85/100=85÷5/100÷5=17/20

Answer:

Mixed numbers

a)  24  1/2  

b) 85/100 = 17/20

Step-by-step explanation:

a) as they asked mixed number we only have to create a fraction for the decimal 0.5 = 1/2 and rewrite 24 1/2 = 24.5

If the question asked for lowest terms 24/5 then 24.5% = 245/1000 = 49/200

b) rewrite 0.85 as a fraction in lowest terms

0.85 = 85% = 85/100 = 17/20

A rectangular portrait is 2 yards wide and 2 yards high. It costs $7.61 per yard to put a gold frame around the portrait. How much will the frame cost?

Answers

Answer:

$60.88

Step-by-step explanation:

2 yards wide and 2 yards high so thats 2 yards per side x 4 is 8, 8 x $7.61 is $60.88

hope this helps:)

a 2 by 2 portrait would look like this

⬜️⬜️
⬜️⬜️

the frame only goes around the edge of the portrait, so we should count how many yards there are on each edge

1. 2
8 ⬜️⬜️ 3
7 ⬜️⬜️ 4
6. 5

then, we multiply the 8 yards by the $7.61.

the answer is $60.88!

simplify

(x^a)^b-c×(x^b)^c-a×(x^c)^a-b

step by step...

Answers

Answer:

= (xa+b)a-b .(xb+c)b-c. (xc+a)c-a

Or

= x(a+b)(a-b). x(b+c)(b-c) .x(c+a)(c-a)

Answer:

[tex]\left(x^a\right)^b-c\left(x^b\right)^c-a\left(x^c\right)^a-b[/tex]

[tex]=x^{ab}-cx^{bc}-a\left(x^c\right)^a-b[/tex]

[tex]=x^{ab}-cx^{bc}-ax^{ac}-b[/tex]

-------------------------

Hope it helps...

Have a great day!!

(02.02 MC)

Tim mixed 2 parts vinegar and 5 parts water to make a cleaning liquid. How many parts of vinegar did Tim mix with each part of water?

one over ten
one over five
two over seven
two over five

Answers

Answer:

2/5 or two over five

Step-by-step explanation:

2:5

Divide each by 5

2/5 : 1

2/5 * 1

2/5

Answer: 2 over 7

Step-by-step explanation:

2/(2+5) = 2/7

Triangle ABC is transformed to obtain triangle A′B′C′:

A coordinate grid is labeled from negative 12 to 0 to 12 on both x and y axes at increments of 1. Triangle ABC has A at ordered pair negative 12, 12, B at negative 9, 6, C at negative 3, 12. Triangle A prime B prime C prime has A prime at ordered pair 4, negative 4, B prime at 3, negative 2, C prime at 1, negative 4.
Which statement is correct for Triangle ABC. and Triangle A prime B prime C prime.? (5 points)

Triangle ABC is similar to triangle A prime B prime C prime. because Triangle A prime B prime C prime. is obtained by dilating Triangle ABC. by a scale factor of 1 over 3. and then rotating it about the origin by 180 degrees
Triangle ABC is similar to triangle A prime B prime C prime., because Triangle A prime B prime C prime. is obtained by dilating Triangle ABC. by a scale factor of 1 over 4. and then rotating it about the origin by 90 degrees
Triangle ABC is similar to triangle A prime B prime C prime., because Triangle A prime B prime C prime. is obtained by dilating Triangle ABC. by a scale factor of 1 over 4. and then rotating it about the origin by 180 degrees
Triangle ABC is similar to triangle A prime B prime C prime. because Triangle A prime B prime C prime. is obtained by dilating Triangle ABC. by a scale factor of 1 over 6. and then rotating it about the origin by 90 degrees

Answers

Answer:

Triangle ABC is similar to triangle A prime B prime C prime., because Triangle A prime B prime C prime. is obtained by dilating Triangle ABC. by a scale factor of 1 over 4. and then rotating it about the origin by 180 degrees

Step-by-step explanation:

Mark branalist
Option 1 is correct
Other Questions
. If R1 = {(x, y)| y = 2x + 7, where x R and -5 x 5} is a relation. Then find the domain and range of R1. find the value of a and be when x is 12 a= 5x^2/2 b= 2x^2(x-5)/10x An analysis of 99 Wall Street traders showed that 32 of their stock picks beat the market average. What is the estimate of the population proportion which of the following is not a feature of natural selection? what is the meaning of multi-party democracy pls help me with thisExpress 0.0065184 in 3 significant figures 1. The first thing I saw in the morning was Henry's van parked in front of the museum. I didn't like it at all, as Henry____ away for another week or so. A. would have been B. would be C. was to be D. should be2. They ____their house yet. They only put it up sales yesterday.A. ought not to sellB. might not be sellingC. needn't have soldD. can't have sold Tm x N: a) 24:(2x-4)+14=26 Dairy cows at large commercial farms often receive injections of bST (Bovine Somatotropin), a hormone used to spur milk production. Bauman et al. (Journal of Dairy Science, 1989) reported that 12 cows given bST produced an average of 28.0 kg/d of milk. Assume that the standart deviation of milk production is 2.25 kg/d. (a) Find a 99% confidence interval for the true mean milk production. Round your answers to two decimal places (e.g. 98.76). Ancient Greeks used myths about their gods primarily toA. explain events in the natural world.B. strike fear in their enemies.C. undermine the Persian religion.D. justify their type of government. Ser - 12 esta noche. El meteorlogo predice que maana al medioda har 25 ms de temperatura. Cul ser la temperatura maana al medioda? Jamie is single. In 2020, she reported $108,000 of taxable income, including a long-term capital gain of $5,800. What is her gross tax liability Last week, Tate went into the hospital for a painful medical procedure. Amazingly, he did not seem worried beforehand, and he showed no signs of autonomic nervous system arousal. In addition, Tate seems to lack a conscience for wrongdoing. It is possible that Tate would be diagnosed as having:Last week, Tate went into the hospital for a painful medical procedure. Amazingly, he did not seem worried beforehand, and he showed no signs of autonomic nervous system arousal. In addition, Tate seems to lack a conscience for wrongdoing. It is possible that Tate would be diagnosed as having: In the context of applications of artificial intelligence (AI), _____ perform well at simple, repetitive tasks and can be used to free workers from tedious or hazardous jobs. Which expression has a value of 15 when n = 7? 1: 43 minus 5 n2: 3 n minus 53: 6 n minus 284: 19 minus StartFraction 28 Over n EndFraction 19. Divide 6/13 by 6/12.A. 12/13B. 13/12c. 1/12D.9/16 25.What effect did the passage of the Jones Act have on the Philippines?A. It granted full independence and self government to the Philippines.B. It promised that the Philippines would ultimately gain its independence.C. It awarded statehood to the Philippines.D. It liberated the Philippines from Japanese occupation. New Mexico was most valuable to both the Union and the Confederacy because ______________ a. it was viewed as fertile farmland. b. it was seen as economically more advanced than California. c. it provided trade access to the west coast and Mexico. d. New Mexico lived up to its reputation as the 'Land of Enchantment.' Help and explain !!!!!! What is the correct expansion of (2x+3) (2x^2-5)